Difference between revisions of "2021 Fall AMC 10B Problems/Problem 5"
(Created page with "==Problem 5== Let <math>n=8^{2022}</math>. Which of the following is equal to <math>\frac{n}{4}?</math> <math>(\textbf{A})\: 4^{1010}\qquad(\textbf{B}) \: 2^{2022}\qquad(\tex...") |
(Redirected page to 2021 Fall AMC 12B Problems/Problem 4) (Tag: New redirect) |
||
(3 intermediate revisions by 2 users not shown) | |||
Line 1: | Line 1: | ||
− | + | #REDIRECT [[2021_Fall_AMC_12B_Problems/Problem_4]] | |
− | |||
− | |||
− | |||
− | |||
− | |||
− | |||
− | |||
− |
Latest revision as of 00:17, 24 November 2021
Redirect to: